Dirichlet's Function: Why is it Difficult to Draw the Graph?

  • Thread starter Thread starter Pyroadept
  • Start date Start date
  • Tags Tags
    Function
Click For Summary
Dirichlet's function is challenging to graph because rational and irrational numbers are densely interspersed, causing continuous transitions between f(x) = 1 and f(x) = 0. This density means that any interval will always contain both types of numbers. Consequently, the lower Riemann sum is always zero, as the minimum vertical area between points is zero, while the upper Riemann sum is always one, reflecting the maximum vertical area. The graph would essentially appear as two solid lines at y=1 and y=0. The discussion emphasizes the implications of the density of rationals and irrationals in the real number line.
Pyroadept
Messages
82
Reaction score
0

Homework Statement


Define Dirichlet's function f by putting f(x) = 1 if x is rational and f(x) = 0 if x is irrational. Explain why it is difficult to draw the graph of f. Prove that the lower Riemann sum L(x_0,...,x_n) is always equal to 0 and the upper Riemann sum U(x_0,...x_n) is always equal to 1.


Homework Equations


Equations for upper and lower Riemann sums.

The Attempt at a Solution


Hi everyone,
Here's what I've done so far:

The graph is difficult to draw because there are infinitely many rational numbers and infinitely many irrational numbers, all interspersed among one another, so you will continuously be switching between f(x) = 0 and f(x) = 1.

For every rational number, there is an irrational number, so any chosen interval will contain both a rational [f(x) = 1] and irrational [f(x) = 0] number.
m = 0 and M = 1
So the lower Riemann sum will be zero, as two points side-by-side (i.e. a rational and an irrational with only 'vertical' area between them) will have minimum vertical area 0.
And, for the upper Riemann sum, two points side-by-side will have maximum vertical area 1.



Is this correct?

Thanks for any help
 
Physics news on Phys.org
Yep, graph difficulty is correct. The graph would look like two solid lines, y=1 and y=0.

The second part is close. Between every two rational numbers lies an irrational number. In fact, the irrationals are dense in the reals. Furthermore, between every two irrationals is a rational, and generally between any two real numbers there are both irrational and rational numbers. And you got the rest.
 
Question: A clock's minute hand has length 4 and its hour hand has length 3. What is the distance between the tips at the moment when it is increasing most rapidly?(Putnam Exam Question) Answer: Making assumption that both the hands moves at constant angular velocities, the answer is ## \sqrt{7} .## But don't you think this assumption is somewhat doubtful and wrong?

Similar threads

Replies
10
Views
2K
  • · Replies 1 ·
Replies
1
Views
1K
  • · Replies 11 ·
Replies
11
Views
2K
  • · Replies 1 ·
Replies
1
Views
2K
  • · Replies 3 ·
Replies
3
Views
1K
Replies
2
Views
2K
  • · Replies 5 ·
Replies
5
Views
2K
  • · Replies 15 ·
Replies
15
Views
2K
  • · Replies 6 ·
Replies
6
Views
1K
  • · Replies 7 ·
Replies
7
Views
1K